Sketch Region Enclosed by f(x) & g(x): Integrate w/ Respect to X or Y

  • Thread starter Thread starter tangibleLime
  • Start date Start date
  • Tags Tags
    Area Curves
Click For Summary
The discussion focuses on sketching the region enclosed by the curves f(x) = 6x - x^2 and g(x) = x^2, and determining whether to integrate with respect to x or y. The integration is set up from 0 to 3, leading to the integral of the difference between the two functions. Participants point out algebraic errors in the calculations, specifically in evaluating the definite integral, which results in incorrect final values. Correcting these mistakes reveals that the final answer should be 9, emphasizing the importance of careful arithmetic in integration problems. The conversation highlights common pitfalls in algebraic manipulation during calculus.
tangibleLime
Messages
71
Reaction score
0

Homework Statement


Sketch the region enclosed by the given curves. Decide whether to integrate with respect to x or y.

symimage.gif



Homework Equations



\int_a^b [f(x) - g(x)]dx


The Attempt at a Solution



f(x) = 6x-x^2
g(x) = x^2

a = 0, b = 3

\int_a^b [f(x) - g(x)]dx

\int_0^3 [(6x-x^2)-(x^2)]dx

((3x^2-\frac{x^3}{3})-(\frac{x^3}{3})\vert_0^3

((3(3)^2-\frac{(3)^3}{3})-(\frac{(3)^3}{3}))

((24)-(3))

21
 
Physics news on Phys.org
nope. you messed up. check your algebra. to make it easier, turn the 2nd line into 6x-2x^2. I am typically always wrong whenever i post on this forum, BUT I got 9
 
Yes, you did mess up on the algebra at the last part.

3 * 3^3 = 27 not 24.

And 3^3/3 = 9, not 3! You have 2 of them so 9 * 2 = 18.

27 - 18 = 9
 
GAH, thanks! I should probably just stick with a calculator to solve the last parts to eliminate errors like that.
 
Question: A clock's minute hand has length 4 and its hour hand has length 3. What is the distance between the tips at the moment when it is increasing most rapidly?(Putnam Exam Question) Answer: Making assumption that both the hands moves at constant angular velocities, the answer is ## \sqrt{7} .## But don't you think this assumption is somewhat doubtful and wrong?

Similar threads

Replies
4
Views
2K
  • · Replies 10 ·
Replies
10
Views
2K
  • · Replies 10 ·
Replies
10
Views
2K
  • · Replies 8 ·
Replies
8
Views
2K
Replies
3
Views
2K
  • · Replies 27 ·
Replies
27
Views
3K
  • · Replies 6 ·
Replies
6
Views
2K
Replies
12
Views
2K
Replies
2
Views
2K
  • · Replies 3 ·
Replies
3
Views
1K